Tải bản đầy đủ (.pdf) (103 trang)

Skkn chuyên đề yếu tố giải tích trong các bài toán đại số, số học, tổ hợp

Bạn đang xem bản rút gọn của tài liệu. Xem và tải ngay bản đầy đủ của tài liệu tại đây (4.3 MB, 103 trang )

Chun đề
YẾU TỐ GIẢI TÍCH
TRONG CÁC BÀI TỐN ĐẠI SỐ, SỐ HỌC, TỔ HỢP
Nguyễn Việt Hà
Trường THPT Chuyên Lào Cai
(Chuyên đề đạt giải nhất)
A. MỞ ĐẦU
Trong chương trình tốn THPT, Giải tích là một nội dung rất quan trọng, học sinh
được học sau khi học Đại số một thời gian dài. Đối tượng nghiên cứu của Đại số và Giải
tích là khác nhau về căn bản. Trong khi Đại số nghiên cứu các đối tượng tĩnh tại, rời rạc
và hữu hạn. Cịn mơn Giải tích nghiên cứu các đối tượng có bản chất biến thiên, liên tục
và vơ hạn. Sự khác nhau căn bản đó dẫn đến những cách tiếp cận khác nhau, lối tư duy
khác nhau khi giải quyết các bài tốn Đại số và Giải tích.
Giải tích là một trong các nội dung luôn xuất hiện trong kỳ thi học sinh giỏi toán
THPT cấp quốc gia (VMO). Tuy nhiên, bên cạnh những câu được phát biểu dưới dạng
giải tích, thì các bài tốn lại được phát biểu dưới dạng đại số nhưng cơng cụ giải tích, kỹ
năng của giải tích giúp chúng ta định hướng tìm ra lời giải. Bên cạnh đó, trong các kì thi
IMO cũng như các kỳ thi học sinh giỏi các nước cũng như khu vực, có nhiều bài tốn cả
đại số, số học cũng chứa đựng các yếu tố giải tích.
Với mong muốn có một tài liệu hệ thống về vấn đề này, tác giả đã quyết định viết chuyên
đề : ‘’Yếu tố giải tích trong các bài tốn đại số, số học, tổ hợp’’. Chuyên đề này đã cố
gắng phân chia các bài toán đại số, số học, và một số bài tốn tổ hợp có thể giải quyết
nhờ cơng cụ giải tích một cách chi tiết nhất có thể với hy vọng giúp ích được cho học
sinh có thêm kỹ năng làm việc với những dạng toán này đồng thời giúp đồng nghiệp có
thêm tài liệu trong giảng dạy học sinh giỏi mơn Tốn cấp quốc gia.
Nội dung của chun đề bao gồm sáu phần trình bày các kĩ thuật giải tích để giải các
bài tốn đại số về phương trình hàm và bất phương trình hàm, các bài tốn về đa thức,
các bài tốn về phương trình và bất đẳng thức, các bài tốn chứng minh tính chất của dãy
số, các bài toán số học, tổ hợp và một số bài toán khác. Các bài toán trong chuyên đề đều
được giải tường minh và đưa ra những nhận xét, bình luận. Phần cuối của chuyên đề là
một số kết luận của tác giả về chuyên đề và những tài liệu tham khảo khi viết chuyên đề


này.
Tác giả của chuyên đề đã làm việc hết sức nghiêm túc, cố gắng tập hợp, sưu tầm, phân
dạng, sắp xếp và liên kết các dạng tốn theo một logic hợp lý nhất có thể theo ý chủ quan
của mình với hy vọng tạo ra được một chuyên đề có chất lượng, có tác dụng đối với các
em học sinh ôn thi VMO và các thầy cơ giáo dạy chun. Tuy đã có nhiều cố gắng nhưng
chun đề khơng thể tránh khỏi những sai sót, thiếu sót. Tác giả rất mong nhận được sự
góp ý của các thầy cơ để chun đề được hồn thiện hơn. Tác giả xin chân thành cảm ơn.

skkn


B. NỘI DUNG
1. KIẾN THỨC CƠ BẢN
1.1. Ánh xạ.
Định nghĩa 1. Ánh xạ f : A  B được gọi là đơn ánh nếu với mọi a, b  A mà a  b thì
f  a   f b   A  B  .
Hệ quả: f là đơn ánh khi và chỉ khi nếu f  a   f  b  thì suy ra a  b (mọi a, b  A )

Định nghĩa 2. Ánh xạ f : A  B được gọi là toàn ánh nếu như với mọi phần tử b  B
đều tồn tại phần tử a  A sao cho f  a   b  A  B  .
Định nghĩa 3. Ánh xạ f : A  B được gọi là song ánh nếu f vừa là đơn ánh vừa là toàn
ánh  A  B  .
1.2. Hàm đơn điệu.
* Hàm số f  x  được gọi là tăng trên  a; b  nếu với mọi x1 , x2   a; b  mà x1  x2 thì
f  x1   f  x2  .
* Hàm số f  x  được gọi là tăng ngặt trên  a; b  nếu với mọi x1 , x2   a; b  mà x1  x2
thì f  x1   f  x2  .

* Hàm số f  x  được gọi là giảm trên  a; b  nếu với mọi x1 , x2   a; b  mà x1  x2 thì
f  x1   f  x2  .

* Hàm số f  x  được gọi là giảm ngặt trên  a; b  nếu với mọi x1 , x2   a; b  mà x1  x2
thì f  x1   f  x2  .

1.3. Hàm số liên tục.
1.3.1. Định nghĩa: Cho hàm số y  f ( x) xác định trên (a,b) và x0 thuộc (a,b). Ta nói
f ( x) liên tục tại x0 nếu lim f ( x)  f ( x0 ) .
x x0

1.3.2 Các tính chất
Tính chất 1. Nếu hàm số f liên tục trên đoạn [a;b] và f (a). f (b)  0 thì có ít nhất một số
c  (a; b ) để f (c)  0 .

Tính chất 2. Giả sử f là một hàm liên tục trên [a;b] và f (a)  A, f (b)  B . Lúc đó nếu C
là một số bất kỳ nằm giữa A và B thì có ít nhất một số c  (a; b) để f (c)  C .
Tính chất 3. Nếu f ( x) liên tục trên (a,b) và x0 thuộc (a, b) sao cho f ( x0 )  0 thì tồn tại
ị  0 sao cho f ( x)  0x  ( x0  ò, x0  ò) ( một lân cận của x0 ).

1.4. Một số kĩ thuật giải tích hay dùng
1.4.1. Với mỗi số thực a cho trước luôn tồn tại một dãy (rn ) các số hữu tỷ sao cho
lim rn = a .

skkn


1.4.2. (Chuyển qua giới hạn một đẳng thức) Nếu hàm số f (x) liên tục tại x0 và
lim xn = x0 thì lim f (xn ) = f ( x0 ).

1.4.3. (Chuyển qua giới hạn một bất đẳng thức) Nếu hàm số f (x) thỏa mãn trên (0; + ¥

)


có k £ f ( x), lim f ( x) = L thì k £ L. ( k < f ( x) thỡ vn ỳng).
xđ + Ơ
1.4.4 Cho hai dóy s thực hội tụ ( xn ),( yn ) . Khi đó nếu xn  yn , n  1,2,... thì
lim xn  lim yn (nếu xn  yn , n  1, 2,... ).
n 

n 

1.4.5 Cho f ( x ) là hàm số biến x , và cho A là một hằng số. Để chứng minh f ( x )  A với
mọi x, ta xây dựng một dãy số (un ) n sao cho f ( x)  un , n ( f ( x)  un , n cũng được) và
lim un  A .
n 

1.5. Bổ sung về yếu tố giải tích của đa thức
1.5.1 Đa thức bậc lẻ ln có nghiệm. Từ đây có thể suy ra các hệ quả sau:
- Nếu P( x ) khơng có nghiệm thì deg P chẵn.
- Nếu P( x ) là đa thức bậc chẵn và có nghiệm duy nhất thì đó phải là nghiệm bội
chẵn.
1.5.2 Đa thức P( x)  ( x  a)k Q( x) thì P( x ) chia hết cho ( x  a ) k 1.
1.5.3 Định lý Rolle. Hàm số f ( x ) liên tục và có a  b để f (a )  f (b)  0 thì tồn tại
c  (a, b) để đạo hàm f (c)  0.
1.5.4 Đạo hàm của đa thức bậc n có đầy đủ nghiệm.
Giả sử x1, x2 ,..., xn là các nghiệm của P( x ) . Khi đó, theo định lí Bézout thì
P( x)  ( x  x1 )( x  x2 )...( x  xn ).
n
P( x ) n 1

Ta có P( x )   ( x  x j ) nên
.

P( x ) i 1 x  xi
i 1 i  j

1.5.5 Đa thức f ( x) có hữu hạn nghiệm thì đạo hàm f '( x) cũng là đa thức nên cũng có
hữu hạn nghiệm. Do đó số điểm cực trị là hữu hạn nên f ( x) sẽ đồng biến, nghịch biến
trên miền đủ nhỏ.
f ( x)
  , tức là với mọi M  0 lớn tùy ý thì
1.5.6 Nếu deg f ( x)  deg g ( x) thì lim
x  g ( x )
tồn tại x để

f ( x)
M.
g ( x)

1.6. Một bổ đề thường dùng
1
1
1
+ k + ... + k với k > 0 và n nguyên dương. Khi đó,
k
1
2
n
Nếu k £ 1 thỡ sn đ + Ơ .

t sn =



Nu k > 1 thì s n bị chặn, tức là có giới hạn hữu hạn.
Chứng minh.
(1) Trước hết, ta chứng minh rằng khi k = 1 thì sn ® + ¥ .

skkn


Thật vậy, bằng đạo hàm, dễ dàng có được ln(x + 1) £ x với mọi x > 0 . Thay x bởi
ta có ln

1
,
n

1
n+1 1
£
hay ³ ln(n + 1) - ln n với mọi n .
n
n
n
n

Suy ra sn ³

å (ln(i + 1) -

ln i ) = ln(n + 1) , mà lim ln(n + 1) = + ¥ nên khng nh
nđ + Ơ


i= 1

ỳng.
T õy d thy rng khng định cũng đúng với mọi k < 1 vì

1
1
>
với k < 1.
k
n
n

Tiếp theo với k > 1 , ta có thể dùng định lý Lagrange:
(2) Xét hàm số f (x ) = x - k với x > 0, k > 1. Chọn hàm số F (x ) =

x 1- k
thì
1- k

F ¢(x ) = f (x ) .

Hàm số F (x ) liên tục trên các đoạn [n , n + 1] và khả vi trên (n , n + 1) nên áp dụng định
lý Lagrange, ta có $ c ẻ (n , n + 1) : F Â(c ) =

F (n + 1) - F (n )
Þ f (c ) = F (n + 1) - F (n ) .
n + 1- n

Chú ý rằng f (c) là hàm nghịch biến nên f (c) > f (n + 1) hay f (n + 1) < F (n + 1) - F (n ) .

Từ đó, cho n = 1, 2, 3,... ta thu được
1
1
1
+ k + ... + k < 1 +
k
1
2
n

n- 1

å (F (i + 1) -

F (i )) = 1 + F (n + 1) - F (1) =

i= 1

k - n 1- k
.
k- 1

1- k

Dễ thấy lim n = 0 vì 1 - k < 0 nên giá trị của s n bị chặn. Bổ đề được chứng minh.
Chú ý rằng với k > 1 thì các dãy đó đều có giới hạn hữu hạn (tăng và bị chặn trên) nhưng
tìm ra chính xác các giá trị đó là điều khơng dễ. Bằng cách tương t, ta cng chng minh
c
1
1

1
1
+
+
+ ... +
đ +Ơ .
2 ln 2 3 ln 3 4 ln 4
n ln n

2. YẾU TỐ GIẢI TÍCH TRONG CÁC BÀI TỐN ĐẠI SỐ, SỐ HỌC
2.1 Kỹ thuật giải tích giải quyết các bài tốn phương trình hàm, bất phương trình
hàm.
2.1.1 Sử dụng dãy số và giới hạn dãy số giải các phương trình hàm, bất phương
trình hàm.
Trong mục này, ta sẽ đi nghiên cứu những bài tốn đại số khơng có gải thiết liên
tục hay khả vi của phương trình hàm, bất phương trình hàm mà có thể sử dụng dãy số và
giới hạn dãy số để giải quyết. Ta hãy bắt đầu bằng ví dụ quen thuộc sau:
Bài tốn 2.1.1.1. Tìm tất cả các hàm số f : ® , đơn điệu trên
và thỏa mãn

f (x + y) = f (x ) + f (y), " x, y Ỵ

(1)

Nhận xét: Nếu bài tốn này, thay giả thiết đơn điệu bằng giả thiết liên tục thì việc giải
quyết là đơn giản Vậy bây giờ khơng cịn yếu tố liên tục nữa, thì ta làm thế nào? Chúng
ta cùng tìm hiểu thơng qua lời giải sau:

skkn



Lời giải. Giả sử f là hàm số thỏa mãn đề bài, khi đó ta có (1). Trong (1), lấy y = x ta
được

f (2 x ) = 2 f (x ), " x Ỵ

(2)

Trong (2), lấy x = 0 ta được f (0) = 0 . Từ (1) và (2) và bằng phương pháp quy nạp, ta
chứng minh được

f (nx ) = nf (x ), " x Ỵ

," n Î

(3)

Trong (1), lấy y = - x và sử dụng f (0) = 0 ta được

f (- x ) = - f (x ), " x Ỵ
Bởi vậy khi n = - 1,- 2, , sử dụng (3) và (4) ta có

(4)

f (nx ) = f (- n (- x ))= - nf (- x ) = nf (x ), " x Ỵ

(5)

Từ (3) và (5) suy ra


f (nx ) = nf (x ), " x Ỵ
Với mọi n = 1,2,

," n Ỵ

(6)

, sử dụng (3) ta có

ỉ 1 ư
ỉ ử
1
m
ỗỗ 1 x ữ
f (x ) = f ỗỗn. x ữ
=
nf
=
m
.
f
x
=
f (x ), " x ẻ
(
)



ỗố n ứ

ỗố n ứữ
n
n

.

Bi vậy

f (rx ) = rf (x ), " x Ỵ

," r Ỵ

(8)

Thay x = 1, ta có f(r) = rf(1), đặt k = f(1) là số thực bất kỳ, ta có f(x)=kx với mọi x thuộc
Q.
(9) .
Trường hợp 1. f là hàm tăng. Ta đã chứng minh được f (x ) = kx, " x ẻ
Vi x ẻ





tựy ý, tn tại hai dãy số hữu tỉ {un }n= 1 , {vn }n= 1 sao cho

un £ x £ vn , " n = 1,2,....; lim un = lim vn = x.
nđ + Ơ

nđ + Ơ


Vỡ f l hm tng nờn kết hợp với (9) ta có

f (un ) £ f (x ) £ f (vn ) Þ kun £ f (x ) £ kvn " n = 1,2,....
Cho n ® + ¥ trong bất đẳng thức trên ta thu được

kx £ f (x ) £ kx Þ f (x ) = kx.
Vậy f (x ) = kx, " x Ỵ

( k là hằng số bất kì). Thử lại thấy thỏa mãn.

Trường hợp 2. f là hàm giảm. Ta chứng minh được f (x ) = kx, " x Ỵ
Với x Î





tùy ý, tồn tại hai dãy số hữu tỉ {un }n= 1 , {vn }n= 1 sao cho

un £ x £ vn , " n = 1,2,....; lim un = lim vn = x.
nđ + Ơ

nđ + Ơ

Vỡ f l hàm giảm nên kết hợp với (9) ta có

f (un ) ³ f (x ) ³ f (vn ) Þ kun ³ f (x ) ³ kvn " n = 1,2,....
Cho n đ + Ơ trong bt ng thc trờn ta thu được


kx ³ f (x ) ³ kx Þ f (x ) = kx.
Vậy f (x ) = kx, " x Ỵ

( k là hằng số bất kì). Thử lại thấy thỏa mãn.

skkn

(9)


Kết luận: hàm số thỏa mãn yêu cầu đề bài là f (x ) = kx, " x Ỵ

( k là hằng số bất kì).

Bình luận: Ta thay tập nguồn và tập đích của bài tốn trên thành (0;+ ¥
thiết đơn điệu, ta có bài tốn sau:
Bài tốn 2.1.1.2. Tìm tất cả các hàm số f :

+

®

+

) và bỏ giả

thỏa mãn

f (x + y) = f (x ) + f (y), " x, y ẻ (0; + Ơ


)

(1)

Li gii. Li giải này nên rút gọn như sau, tận dụng BT ở trên.
Ta có f ( y) > 0 với mọi y > 0 nên f ( x + y) > f ( x ) . Đặt z = x + y > x thì
f ( z) > f ( x ) với mọi z > x nên có ngay f ( x ) là hàm đơn điệu tăng. Sử dụng kết quả ở
trên ta có f ( x ) = kx .
Nếu bài tốn đổi thành - ® - thì hồn tồn tương tự, sẽ có đơn điệu giảm (và
đây là ý cơ bản nhưng có vẻ trước giờ khơng có tài liệu nào đề cập).
Bình luận: Trong bài tốn 2.1.1.2, ta đã bỏ giả thiết đơn điệu, nhưng cuối cùng ta lại có
thể chứng minh được tính đơn điệu, nhờ đó mà đường lối giải lại quay về như bài toán
2.1.1.1. Từ bài toán 2.1.1.1, ta giữ lại tập nguồn và tập đích là , bỏ giả thiết đơn điệu,
và thêm giả thiết hàm nhân tính, ta có bài tốn sau:
Bài tốn 2.1.1.3. Tìm tất cả các hàm số f : ®
thỏa mãn đồng thời hai điều kiện sau


f (x + y) = f (x ) + f (y), " x, y Ỵ



f (xy) = f (x ) f (y), " x, y Ỵ

(1)
(2)

Lời giải. Từ (1), ta chứng minh được các kết quả sau

ìï f (rx ) = rf (x ), " x Ỵ , r Ỵ

(3)
ï
í
ïï f (0) = 0, f (- x ) = - f (x ), " x ẻ
(4)

2
T (2) cho y = x ta được f (x 2 ) = éëf (x )ù
û , " x Ỵ . Suy ra f (x ) ³ 0, " x ³ 0 .
Đến đây vẫn suy ra đồng biến tương tự trên. Suy ra f ( x ) = kx , thử lại được
k = 0 hoặc k = 1 .
Bài tốn được giải hồn tồn.
Bình luận: Một lần nữa, trong bài toán 2.1.1.3, ta bỏ giả thiết đơn điệu nhưng thêm giả
thiết hàm nhân tính, ta lại chứng minh được hàm đồng biến nên lại có thể giải tiếp giống
bài tốn 2.1.1.1. Tuy nhiên điểm chung của cả ba bài tốn trên là đều có giả thiết cộng
tính.
Sau đây ta đi xét những bài tốn phức tạp hơn:
Bài toán 2.1.1.4. [China MO 1998] Cho hàm số f : ®
thỏa mãn đồng thời các
điều kiện
(i)

ỉx ư
f 2 (x ) Ê 2 x 2 f ỗỗ ữ
"x ẻ

ỗố 2 ữ


(ii)


f (x ) Ê 1 " x ẻ (- 1;1).

x2
"x Ỵ
Chứng minh rằng: f (x ) £
2

;

.

skkn


Lời giải. Từ giả thiết (i) thay x = 0 thì ta có f (0) = 0 , đúng với iu kin, nờn ta ch

ổ ử

ỗố 2 ứ

x
cn chng minh cho trường hợp x ¹ 0 . Với mỗi x ¹ 0 ta có f 2 (x ) £ 2 x 2 f ỗỗ ữ
ữ.
ổx ử
ỗỗ ữ
2
f

ổf (x )ử


ốỗ 2 ứ



ị ỗỗ 2 ữ Ê


ỗố x ứ
x2

ổx ử
ỗỗ ữ
2
f

ổ2 f (x )ử

ỗ2 ứ


ỗỗ

Ê
.
2
ỗ 2 ữ

ổx ữ


ốỗ x ứ
ỗỗ ữ
ỗố 2 ÷
ø
2 f (x )
xác định bởi công thức g (x ) =
" x ạ 0 . Ta cú
x2
ổx ữ

ỗỗ ữ
2
2
f
ổ2 f (x )ữ


ỗố 2 ứ
ổx ử
2
ỗỗ
ỗỗ ữ

Ê

g
x
Ê
g
(

)


2
ỗỗ x 2 ữ

ỗố 2 ứ
ổx ữ



ỗỗ ữ

ỗố 2 ứ
2

Xột hm g :

*

đ

2

nờn bng phng phỏp quy np ta c

ổx ử
ổx ữ

n

2n g ỗ
g 2 (x ) Ê g ỗỗ n ữ

g
x
Ê
" n ẻ *.
(
)



n

ỗố2 ữ


ố2 ứ
2
Li cú f (x ) Ê 1 " x Ỵ (- 1;1) nên g (x ) £ 2 " x Ỵ (- 1;1) \ {0}. Do đó
x
2 n+ 1

g (x ) £

2n

2

2 n+ 1


x

2

=

2n

2

1

x2

"nỴ

*

n- 1

:

x
Ỵ (- 1;1)
2n

2 n+ 1

ị g (x ) Ê lim


2

2n

x

2 n- 1

1

nđ + Ơ

Vy ta có f (x ) £

= 1 " x ¹ 0 c nh ị f (x ) Ê

x2
"x ẻ
2

x2
" x ¹ 0.
2

.

Tiếp theo, ta xét một bài toán tổng quát hơn:
Bài tốn 2.1.1.5. [Olympic sinh viên tồn quốc 2016, mơn Giải tích].
Cho a  1 là một số thực và hàm f : R  R thỏa mãn đồng thời hai điều kiện

•  f  ax    a 3 x 2 f  x  với mọi số thực x .
2

• f bị chặn trong một lân cận nào đó của 0.
Chứng minh rằng f  x  

x2
với x  R.
a

Lời giải. Trong i. thay x  0 ta thu được

 f  0    0  f  0   0.
2

Với mọi x  0 , từ i. ta có
 f  ax  
f  x   3 2   0
ax
2

skkn


với x  0. Từ đó, ta được

f  x  0

với mọi x  R . Nếu a  1 thì từ i. ta được
 f  x    x 2 f  x  .

Từ đây, ta suy ra f  x   x 2 với mọi x  R . Như vậy, bất đẳng thức cần chứng minh đúng
2

với trường hợp a  1 .
Bây giờ, ta sẽ xét trường hợp a  1 . Đặt
g  x 

f  x
2

x
a

với mọi x  0 thì
f  x 



f  x
0
x2
a

x2
g  x  , x  0.
a

Khi đó, từ i. ta suy ra
2


2
  ax 2

3 2 x


g  ax   a x
g  x  , x  0
 a

a



  g  ax    g  x  , x  0.
2

(1)

Ta sẽ chứng minh
 x 
g  x  g  n 
a 

2 n

, x  0, n  N*

(2)


bằng phương pháp quy nạp toán học theo n . Thật vậy, trong (1) thay x bởi
1
2

  x 
g  x    g   ,
  a 

x
ta được
a

x  0.

(3)

Như vậy, mệnh đề (2) đúng với n  1 . Giả sử mệnh đề đúng với n  k  2 tức là
2 k

 x 
g  x   g  k  , x  0.
a 
x
Trong (4) thay x bởi ta được
a
 x
 x 
g    g  k 1 
a
a 


(4)

2 k

, x  0.

(5)

, x  0.

(6)

Từ (3) và (5) , ta suy ra
 x 
g  x   g  k 1 
a 

 k 1
2  

Do đó, mệnh đề (2) đúng với n  k  1 . Theo nguyên lý quy nạp toán học mệnh đề (2)
đúng với mọi n  N* . Như vậy, ta có
2 n

 x 
g  x  g  n  ,
a 
*
với x  0 , n  N . Từ định nghĩa của hàm g ta thu được


skkn


2 n




 







 x 
f  n 
2 n
 x 
a  .
(7)
g  x  g  n 
2 
a 
 x  
 n
a  


a 
x
Vì x  0 và a  1 nên với n đủ lớn thì n sẽ thuộc một lân cận nào đó của điểm 0 . Do
a
đó, từ ii. ta suy ra tồn tại n0  N* và M  0 sao cho với n  n0 ta có
 x
f n
a


  M.


Kết hợp với (7) ta được

 x 
g  x  g  n 
a 

Theo quy tắc L’Hospital thì

2 n




 








 x 
f  n 
a 
2 
 x  
 n
a  

a 

2 n

2 n 1



a

2n
1n

x2

n

M2 .


(8)

2n  1
 0.
2n
n 

lim

Do đó
2 n 1

lim
n 

a

2n

n

1n

x2

M 2  1.

Từ (8) cho n   , ta thu được g  x   1 với mọi x  0 hay
f  x

 1, x  0.
x2
a

Từ đây, ta có

f  x 

x2
, x  0.
a

Chú ý rằng f  0   0 và f  x   0 nên ta được

f  x 

x2
,
a

với x  R . Bài tốn được chứng minh.
Binh luận:
• Bài tốn trên là một bài tốn về bất phương trình hàm có sử dụng tính chất giải tích và
khá nặng về mặt kĩ thuật. Để chứng minh f  x   P  x  về mặt ý tưởng ta tìm một đánh
giá f  x   P  x  .un với lim un  1 hoặc f  x   P  x   un với lim un  0 , trong bài toán này
n 

n 

thì


skkn


2 n 1

un 

a

2n
1n

x2

M2

n

và lim un  1. Khi tìm được đánh giá (13), thì ta thấy rằng lim

n 

n 

x
 0 với mọi a  1 nên ta
an

có thể sử dụng giả thiết ii. của bài toán để tiếp tục đánh giá. Bài toán trên là bài toán tổng

quát của bài tốn 2.1.1.4
• Ngồi cách giải đã trình bày ở trên, ta có thể tiếp cận bài tốn theo hướng khác như
sau:
Vẫn như lời giải ở trên, ta chứng minh được
f  0   0, f  x   0
với mọi x và bất đẳng thức cần chứng minh đúng khi a  1 và chỉ còn xét trường hợp
a  1 . Trong i. thay x bởi

x
ta được
a

2
x
 f  x    ax 2 f   , x  R \ 0.
a
2
Giả sử rằng tồn tại z  0 sao cho f  z   z . Từ ii. ta lần lượt có:

z 1
f    2 z2
 a  az

 

2

z2
 ;
a

2

2
1 z 
z2
 z 
f  2 
  .
2 
a
a 
 z a 
a 
a

Từ đó ta được
1
 z 
f  3
 az 2 .
2
a 
 z 
a 2 
a 

Bằng phương pháp quy nạp toán học, ta có:
 z 
f  n   a 2 n 5 z 2 , n  2.
a 


(20)

Do điều kiện thứ hai của bài toán, vế trái của (20) bị chặn trên, trong khi vế phải lớn tùy
ý khi n đủ lớn, vơ lý! Do đó điều giả sử là sai hay
f  x   x 2 , x  R \ 0.
(21)
Xét hàm số
x2
h  x  f  x 
 f  x
a

thì từ i. ta được
2


x2 
x2 
2  x
h
x


ax
h

, x  R \ 0.
  


  
3 
a
a
a







Điều này tương đương
x2
x
 h  x    2 h  x   ax 2 h   , x  R \ 0
a
a
2

Từ đó, ta được

skkn


x2
a2  x 
x
h  x   ax 2 h    h  x   h   ,
a

2 a
a
bất đẳng thức này đúng cả khi x  0 vì h  0   0 . Bằng phương pháp quy nạp tốn học ta
2


n

 a2   x 
h  x     h  n  , n  N.
 2  a 

(22)

Từ (21) và (22) ta được
n

 a2   x
h  x    h  n
 2  a

n

2
 a   x

   f n
  2  a



.


Do đó, ta có
n

 a2   x 
x2
h  x      n   n , n  N.
2
 2  a 
2

Vì n có thể lớn tùy ý nên điều này chỉ đúng khi và chỉ khi h  x   0, x  R . Từ đây, ta
suy ra điều phải chứng minh.
Bài tốn 2.1.1.6. Tìm tất cả các hm f : đ
tha món vi mi x ẻ
ta cú


f (2 x ) = 2 f (x )



f (x )- x £ 1

(1)
(2)

Lời giải. Từ (1), bằng cách sử dụng quy nạp, ta chứng minh được


f (2n x ) = 2n. f (x ), " x Ỵ
Bây giờ, ta sẽ chứng minh f (x ) = x, " x Ỵ

,n Î

(3)

. Thật vậy, giả sử có x0 Î

sao cho

f (x0 ) ¹ x0 . Đặt f (x0 ) = x0 + e với e ¹ 0 . Khi đó, ta có
f (2n x0 ) = 2n. f (x0 ) = 2n x0 + 2n e, " n Ỵ

.

Suy ra

f (2 n x0 )- 2 n x0 = 2 n. e , " n Ỵ

(4)

n
n
Do (2) nên f (2 x0 )- 2 x0 £ 1 và như thế, kết hợp với (4), ta thu được

2 n. e £ 1, " n Î

(5)


Tuy nhiên, kết quả này không thể nào được thỏa mãn với mọi n Ỵ . Mâu thuẫn thu
được cho ta kết quả vừa khẳng định ở trên, tức f (x ) = x, " x Ỵ . Hàm này thỏa mãn
các u cầu của bài tốn.
Nhận xét. Có thể thay số 2 trong (1) bởi số thực dương khác 1 và VP = 1 trong (2)
bởi các số thực dương bất kỳ.
Bài toán 2.1.1.7 [VMO, 2003-A]. Gọi Á là tập hợp tất cả các hàm số f : + ® + thỏa
mãn f (3x ) ³ f ( f (2 x ))+ x, " x Ỵ

+

(1)

Tìm hằng số A lớn nhất để với mọi f Ỵ Á và với mọi x > 0 ta đều có

f (x ) ³ Ax.
Lời giải.

skkn


Trong (1) , thay x bởi

x
ta có
3

ỉ ỉ2 x ư÷
ư x
f (x ) f ỗỗ f ỗỗ ữ

+ , "x ẻ


ữữ
ỗố ỗố 3 ứ
ứ 3
ổ ổ2 x ửữ

T (2), do f ỗỗ f ỗỗ ữ
> 0 nờn ta suy ra


ỗố ỗố 3 ữ
ứữ


f (x )>

x
, "x ẻ
3

+

+

(2)

(3)


1
. Khi ú, từ (2) và (3), ta có
3
ỉ2 x ÷
ư x
x 2 a12 + 1
2 2x

f (x ) > a1 . f ỗ ữ
+ > a1 . + =
x, " x ẻ
ỗố 3 ÷
ø 3
3
3
3
2 a12 + 1
Bằng cách đặt a2 =
và áp dụng (4) vào (2) ta có
3
ỉ2 x ÷
ư x
x 2 a22 + 1
2 2x

f (x ) > a2 . f ỗ ữ
+ > a2 . + =
x, " x ẻ
ỗố 3 ữ
ứ 3

3
3
3
t a1 =

+

+

(4)

(5)

C th, bng cỏch lp lại các quy trình đánh giá giống nhau như vậy, ta thu được
f (x ) ³ an x
(6)
với mọi x Î

+

và với mọi n Î

*

, trong đó (an ) là dãy truy hồi được xác định bởi

2 an2 + 1
1
với mọi n ³ 1 .
a1 = và an+ 1 =

3
3
Ta chứng minh được (an ) là dãy tăng và bị chặn trên bởi

1
nên có giới hạn hữu hạn. Từ
2

1
.
2
Bây giờ, trong (6) ta cho n đ + Ơ thỡ c
ú dễ dàng tìm được lim an =

f (x ) ³
Mặt khác, dễ thấy hàm số f (x ) =

(lim an ) x =

1
x, " x Ỵ
2

+

.

1
1
x thỏa mãn điều kiện đề bài. Vậy Amax = .

2
2

Tiếp theo ta xét một bài toán gần gũi với bài trên:
Bài toán 2.1.1.8 (Bulgaria, 2008) Tìm tất cả các hàm số f :

®

f (x + y 2 ) ³

(1)

(y + 1) f (x )

thỏa mãn

với mọi cặp số thực x, y .
Lời giải. Thay y = - 1 và thay x bởi x - 1 vào (1) ta thu được f (x ) ³ 0 với mọi

xỴ

. Từ đây, kết hợp với (1) ta suy ra

f (x + y) ³

(

)

y + 1 f (x ) ³ f (x )


skkn


với mọi x Ỵ
và với mọi y ³ 0 . Kết quả này chứng tỏ f là hàm không giảm trên
Bây giờ, ta viết lại (1) dưới dạng

f (x + y2 )- f (x ) ³ yf (x )
với mọi x, y Ỵ

.

(2)

, và xét hai hãy (an ),(bn ) với a0 = b0 = 0 ,

1
+
22

1
1
1
,
b
=
1
+
+

+
n
2
n
n2
1
Trong (2), ta thay x bởi x + ak và thay y bởi
thì có
k+ 1
f (x + ak ) f (x )
f (x + ak + 1 )- f (x + ak ) ³
³
k+ 1
k+ 1
với mọi x Ỵ
và k Ỵ
. Trong bất đẳng thức trên, lần lượt cho k = 0,1,
an = 1 +

+

, n - 1 và

cộng tất cả các bất đẳng thức lại theo vế, ta thu được

f (x + an )- f (x ) ³ bn f (x )
với mọi x Ỵ
và n Ỵ
. Mặt khác, ta dễ dàng chứng minh c an < 2 v
bn đ + Ơ (õy là hai kết quả quen thuộc, bạn đọc có thể tự chứng minh). Do đó, từ bất

đẳng thức trên, ta có

f (x + 2)- f (x ) ³ f (x + an )- f (x ) ³ bn f (x )
với mọi x Ỵ

và n Ỵ

. Nếu tồn tại x0 Î

sao cho f (x0 ) > 0 thì từ bất đẳng thức

trên, bằng cách thay x = x0 và cho n đ + Ơ , ta s thu c iu mâu thuẫn (vế phải
tiến đến dương vô cùng trong khi vế trái là hằng số). Do vậy, ta phải có f (x ) = 0 với
mọi x Ỵ
. Dễ thấy hàm này thỏa mãn các yêu cầu của đề bài.
Bài tốn 2.1.1.9. Cho hàm số f : + ®
thỏa mãn đồng thời các điều kiện
(1) f tăng thực sự trên

+

;

æ 2 xy ÷
ư f (x ) + f (y)
với mọi x, y ẻ


ỗố x + y ữ
2



(2) f ỗỗ

+

.

Chng minh rằng tồn tại x0 > 0 sao cho f (x0 ) < 0 .
Lời giải. Giả sử kết quả cần chứng minh là sai, tức ta có f (x ) ³ 0 với mọi x > 0 . Thay

y=

x
vào (2) ta c
3

ổx ử
ổx ử
+
ỗỗ ữ
2. f ỗỗ ữ

f
x
+
f
,
"
x


.
(
)



ỗố 2 ữ

ốỗ 3 ø
Tiếp tục, thay y = 2 x vào (2) ta được
ỉ4 x ư f (x ) + f (2 x )
f ỗỗ ữ
.


ỗố 3 ứ
2
T ú suy ra

skkn

(3)


ổx ử
f ỗỗ ữ

ữ+
ỗố 4 ứ

ổx ử
f ỗỗ ữ


ỗố 3 ữ

2

ổx ử
f ỗỗ ữ


ốỗ 2 ứ
, "x ẻ

+

(4)

Kt hp hai kết quả (3) và (4) lại, ta thu được

ỉx ư
f çç ÷
+
÷
çè 4 ÷
ỉx ÷
ư
ø
2. f çç ÷

÷³ f (x ) +
ỗố 2 ứ
2

ổx ử
f ỗỗ ữ

ỗố 2 ữ

,

Hay


ổx ử 1 ộ ổx ử
ổx ử
ỗỗ ữ
ờf ỗỗ ữ

f (x )- f ỗỗ ữ
Ê
f



ữ 2 ờ ốỗ 2 ứ
ữ ốỗ 4 ứ
ữỳ, " x ẻ
ỗố 2 ứ




+

(5)

T (5), bng cỏch lp lại các quy trình đánh giá, ta có

ỉx ư 1 ộ ổx ử
ổx ửữự
ỗỗ ữỳ
ờf ỗỗ ữ
f (x )- f çç ÷
£
f
÷
÷
÷ 2 ê çè 2 ø
÷
çè 2 ø
èç 4 ứữỳ


ổ x ửữự
1 ộ ổx ử
ỗỗ ữỳ
Ê 2 ờf ỗỗ 2 ữ
f





ốỗ23 ứữỳ
2 ở ố2 ứ

...................................
ổ x ửữự
1 ộ ổx ử
ỗỗ
Ê n ờf ỗỗ n ữ
f

ữỳ

ốỗ2 n+ 1 ứữỳ
2 ờở ỗố2 ứ

Ê

1
2n

ổx ử
f ỗỗ n ữ

ữ, " x ẻ
ỗố 2 ø

+


,n Ỵ

*

.

(6)

( f ( x ) ³ 0 " x > 0 f(x) >= 0 với mọi x > 0).

x
x
³ n và f là hàm tăng thực sự nên từ (6), ta suy ra
2 2
ổx ử 1 ổx ửữ
ỗ , " x Ỵ + , n Ỵ *.
0 < f (x )- f ỗỗ ữ
(7)

ữÊ 2 n . f ốỗỗ 2 ứữ

ỗố 2 ứ
1
Trong (7), c nh x , ta cho với để ý lim n = 0 thì được
2
ỉx ử
0 < f (x )- f ỗỗ ữ
Ê 0, " x ẻ .

ỗố 2 ữ


Mt khỏc, do x >

Mõu thun này chứng tỏ điều giả thiết là sai. Nói cách khác, phải tồn tại ít nhất một số
x0 > 0 sao cho f (x0 ) < 0 .
Bài toán 2.1.1.10 [Chọn đội tuyển Sư phạm, vòng 2, 2013]. Cho hàm số f xác định
trên đoạn [0;2013] và nhận các giá trị trên tập

thỏa mãn đồng thời các điều kiện sau

(1) f (x ) £ 0, " x Ỵ [0;2013] và f (2013) = - 2013;

skkn


(2) Bất đẳng thức f (x1 ) + f (x2 ) +

+ f (xn ) ³ f (x1 + x2 +

, xn Ỵ [0;2013] sao cho

mọi số ngun dương n và n số thực x1 , x2 ,

x1 + x2 +

+ xn ) đúng với

+ xn £ 2013 .

Chứng minh rằng f (x ) ³ - 2 x, " x Î [0;2013].

Lời giải. Trong (2), ta xét n = 2 và thay x1 = 0, x2 = 2013 thì thu được f (0) ³ 0 . Từ
đó, kết hợp với (1), ta có ngay f (0) = 0 . Kết quả này cho thấy khẳng định bài toán đúng
với x = 0 và như thế ta chỉ còn phải chứng minh kết quả bài tốn cho trường hợp
x Ỵ (0;2013] nữa là đủ.
Tiếp theo, ta cũng xét điều kiện (2) với trường hợp n = 2 và thay x1 = x, x2 = y - x
với 0 £ x £ y £ 2013 thì có

f (y) £ f (x ) + f (y - x ) £ f (x ), " x, y Ỵ [0;2013], x £ y

(3)

Từ (3), ta suy ra ngay f là hàm không tăng trên [0;2013]. Trong (2), ta cho n = 2 k với

kỴ

2013
thì được
2k
ỉ2013ư
ỉ k 2013ửữ
ỗỗ2 .
2 k. f ỗỗ k ữ

f

ữ= f (2013) = - 2013 .

ỗố 2 ứ
ốỗ
2 k ứữ


v thay x1 = x2 =

= x2k =

Và như thế, ta có

ỉ2013 ư
2013
f ỗỗ k ữ

, "k ẻ


ỗố 2 ứ
2k

(4)

2013
= 0 nờn ta có thể chia (0;2013] thành hợp của vơ số đoạn, mi on cú
kđ + Ơ
2k
ộ2013 2013 ự
ộ2013 2013 ự
dng ờ m + 1 , m ú. Bây giờ, sử dụng hai kết quả (3) và (4), với mỗi x Ỵ ê m + 1 , m ú
êë2
êë2
2 ú
2 úû

û
Vì lim

ta cú

ổ2013ử
2013
2013
f (x ) f ỗỗ m ữ
= - 2. m + 1 - 2 x,

m

ỗố 2 ứ
2
2

(5)

Bt đẳng thức (5) cho ta kết quả của bài toán.
Bài tốn 2.1.1.11 [Romania TST, 2007]. Tìm tất cả các hàm f :
2

f (x )- f (y) £ (x - y) , " x, y ẻ

đ

(1)

x+ y

vo (1), ta thu c
2
2
ổx + y ữ
ử (x - y)
f (x )- f ỗỗ
, " x, y ẻ
(2)

ữÊ
ỗố 2 ứ
4
x+ y
Tng t, thay x bởi
vào bất phương trình (1) ta cũng có
2
Lời giải. Thay y bởi

skkn

thỏa mãn


ổx +
f ỗỗ
ỗố 2

2

(x - y)

yử

f
y
Ê
, " x, y ẻ
(
)



4

(3)

Cng hai bất đẳng thức (2) và (3) lại theo vế và sử dụng bất đẳng thức trị tuyệt đối
a + b ³ a + b , ta có
2

f (x )- f (y) £

(x - y)
2

, " x, y Ỵ

(4)

Từ đây, bằng cách lặp lại quy trình tương tự như trên, ta chứng minh được
2


f (x )- f (y) £

(x - y)

, " x, y Ỵ

,n Ỵ

+

(5)
2
Trong (5), cố định x, y ta cho n đ + Ơ thỡ c f (x )- f (y) £ 0 , tức
n

f (x ) = f (y), " x, y Ỵ
Từ (6) suy ra f (x ) º c, " x Ỵ

(6)

. Thử lại, ta thấy thỏa mãn.

Bài tốn 2.1.1.12. Tìm tất cả các hm s f : [1; + Ơ ) đ [1; + ¥ ) và thỏa mãn:
i. f (x ) ³ x với mọi x ³ 1.
ỉf (x ) ÷
ư
2
÷
ii. f ççç

÷ = x với mọi x ³ 1.
÷
çè x ø

Lời giải.
f ( x)
, ta có
x
f ( x)
 f ( x)  f ( x)
f
 x2 
 f ( x)  x3 .

x
x
x



Trong điều kiện i, thay x 

Giả sử ta đã có f ( x)  x thì dễ dàng biến đổi được f ( x)  x
a

Tương tự, nếu có f ( x)  x b thì ta cũng có f ( x)  x

b2
b


a2
a

.

.

u1  1, u2  3,

Từ đó, xét dãy số 
và bằng quy nạp, ta chứng minh được
un  2
u

,
n

1,
2,3,...
n

1

un


xu2 k 1  f ( x)  xu2 k với mọi k nguyên dương.
3
Hơn nữa, ta chứng minh được un  với mọi n  1 nên
2

n
un  2 2
2
un 1  2 
 un  2  ...    u1  2 .
un
3
3

Từ đây suy ra lim un  2 và theo nguyên lý kẹp, ta phải có f ( x)  x 2 với mọi x  1 .
Thử lại ta thấy thỏa, vì chú ý rằng f ( x)  x 2  x với mọi x  1.
Nhận xét. Bài tốn cũng đúng khi thay miền thành



tức là: Tìm f :



sao

 f ( n) 

2
cho f 
n .
n


Bài toán 2.1.1.13.[Belarus MO 1997] Cho hàm số f :


skkn

+

®

+

thỏa mãn điều kiện


f (2 x ) ³ x + f ( f (x ))" x Ỵ
Chứng minh rằng f (x ) ³ x " x Ỵ

+

Kết hợp với f (x ) Ỵ
Đặt a 1 =

"x Ỵ

+

1
, với mỗi x Ỵ
2

+


+

.

+

.

.

Lời giải.
Từ giả thiết f (2 x ) ³ x + f ( f (x ))" x Ỵ

f (x ) ³

+

+

suy ra

ỉ ỉx ửử
x
ữữ
+ f ỗỗ f ỗỗ ữ

ữ" x ẻ




2
ố ố 2 øø
x
ta có f (x )>
"x Ỵ
2

+

.

ta có f (x )> a 1 x suy ra

1 + a 12 ) x
æ ổx ửử
(


x
x
x

ỗ ữ
f (x )
+ f ỗỗ f ỗỗ ữ
.



ữ> 2 + a 1 f ốỗỗ 2 ứ

ữứ
ữ>
ỗố ỗố 2 ø
2
2

1 + a 12
ta có f (x )> a 2 x " x > 0 .
2
Bằng phương pháp quy nạp ta có f (x )> a n x " n Ỵ
Đặt a 2 =

*

, ở đây dãy số (a n ) xác định

bởi:

ìï
1
ïï a 1 = ;
ïï
2
í
ïï
1 + a n2
"nẻ
ùù a n+ 1 =
2
ùợ

M lim a n = 1 nên từ f (x )> a n x " n ẻ

*

nđ + Ơ

+

Bi toỏn 2.1.1.14. Tỡm tt c cỏc hm f :
(i)

f là hàm số nghịch biến trên

+

;

(ii)

f (2 x ) = 2016

+

.

- x

f (x ) " x Ỵ

Lời giải.

- x
Ta có f (2 x ) = 2016 f (x ) " x ẻ
Xột hm s h :

+

đ

+

f (2 x ) = 2016- x f (x ) " x ẻ
+

đ

Do g (t + 1) = g (t ) " t Î
chặn trên

+

ta có f (x ) ³ x " x Î

®

+

+

.


thỏa mãn đồng thời các điều kiện

Û 20162 x f (2 x ) = 2016 x f (x )" x Î

x
xác định bởi công thức h (x ) = 2016 f (x ) " x Ỵ

+

Suy ra h (2 x ) = h (x ) " x Ỵ

Xét tiếp hàm g :

+

*

+

+
+

.

.

. Do x = 2 t Û t = log 2 x , thay vào đẳng thức
+

ta được 20162


t+ 1

t

f (2t + 1 ) = 20162 f (2t ) .
t

xác định bởi công thức g (t ) = 20162 f (2t ) " t Ỵ

.

nên g (t ) là hàm tuần hồn với chu kì T = 1 , do đó g (t ) bị

x
, suy ra hàm số h (x ) = g (log2 x ) = 2016 f (x ) bị chặn trên

- x
số f (x ) = 2016 g (log2 x ) cũng bị chặn trên

skkn

+

.

+

và hàm



ổx ử

ổx ử
*
= h ỗỗ n ữ
nờn vi x > y > 0 thỡ

ữ" n ẻ
ỗố2 ứ
x
y
ổx ử
ổy ử
2n
2n


ỗỗ ữ
h (x )- h (y) = 2016 f ỗ n ữ
2016
f



ỗố2 ứ
ốỗ2 n ứ
x
y ử


ổ ổx ử

ổy ử


2n
2n ữ
ỗỗ f ỗỗ ữ


ị 2 (h (x )- h (y)) = ỗỗ2016 - 2016 ữ
+
f

ỗỗ n ữ

nữ










ỗố
2
2




x
y ử

ổ ổx ử

ổy ử


2n
2n ữ
ỗỗ f ỗỗ ữ


+ ỗỗ2016 + 2016 ữ
f
"nẻ

ỗỗ n ữ

nữ

ữữ








ỗố
2
2




Ta cú h (x ) = h ỗỗ ữ

ữ=
ỗố 2 ứ

hay 2 (h (x )- h (y)) = an + bn " n ẻ

*

*

, õy

x
y ử

ổ ổx ử

yử



2n
2n ữ
ỗỗ f ỗỗ ữ
ữ+ f ổ


an = ỗỗ2016 - 2016 ữ


ỗỗ n ữữ> 0 .

nữ






ỗố
2
2 ứứ


Kt hp vi hm s f b chn nờn ta có lim an = 0 . Lại có hàm s f nghch bin nờn

nđ + Ơ

x
y ử


ổ ổx ử

ổy ử

ữ< 0 .
2n
2n ữ
ỗỗ f ỗỗ ữ


bn = ỗỗ2016 + 2016 ữ
f


ỗỗ n ữữ

nữ




ố2 ứứ
ỗố
ứố ố2 ứ

M

a1 + b1 = a2 + b2 =

= an + bn =


nên

lim (an + bn ) = a Ê 0

hay

nđ + Ơ

h (x ) £ h (y). Mặt khác, do h (x ) = h (2 x ) = = h (2n x ) = " x Ỵ + , n Ỵ
é
x

n
n+ 1
với n = êlog2 ú thì 2 y £ x £ 2 y và n £ log2 < n + 1, suy ra
êë
y

û
n+ 1
n
h (2 y) £ h (x ) £ h (2 y) Þ h (y) ³ h (x ) ³ h (y) Þ h (x ) = h (y)

*

nên

hay h (x ) º c > 0 .
- x

Kết hợp với f (x ) = 2016 h (x ) " x Ỵ

ta có f (x ) =

+

c
"x Ỵ
2016 x

+

.

Bài tốn 2.1.1.15. Tìm tất cả các hàm số f : ¡ ® ¡ thoả mãn đồng thời các điều kiện:
f ( x  y )  f ( x )  f ( y ), x, y 

(1)

f ( x )  e x  1, x 

(2)

Lời giải.
 f ( x  0)  f ( x )  f (0)  f (0)  0

Theo giả thiết ta có: 

0
 f (0)  e  1  f (0)  0


 f (0)  0

Từ (1) ta có: 0  f (0)  f ( x  (  x))  f ( x )  f (  x )  f ( x )  f (  x )  0, x 
Từ (1) và (2) ta có:

skkn

(3)


x 

f ( x)  f  2n. n   2n
 2 

x

n
 x 
f  n   2n (e 2  1), x 
2 

x

 f ( x)  x.

e 1
, x  0, n 
x

2n
2n

*

(4)

ex  1
 1 nên từ (4) cho n   ta được f ( x )  x, x  0.
x 0
x

Do lim

Kết hợp với f (0)  0 ta được f ( x )  x, x 

(5)

Vì thế f ( x )  f (  x )  x  (  x )  0, x  . Kết hợp với (3) ta được
f ( x )  f (  x )  0, x 

Do đó với mọi x 

ta có f (  x )   x   f ( x )   x  f ( x )  x

Kết hợp với (5) suy ra hàm số cần tìm là f ( x )  x, x  . Thử lại thấy thỏa mãn.
Bài tốn 2.1.1.16. Tìm số thực k lớn nhất để nếu f ( x ) là hàm số tùy ý xác định trên
thỏa mãn bất phương trình hàm:

3 f ( x)  3 f ( x) 


9 4
f ( x )  1, x 
4 3

(*)

thì ta ln có f ( x )  k , x 
Lời giải
Giả sử tồn tại số thực k thỏa mãn bài tốn.
4
4
Khi đó vì f ( x )  , x  thỏa mãn bất phương trình hàm (*) nên ta có  k .
3
3
Tiếp theo, ta sẽ chứng minh rằng: Nếu f là hàm số tùy ý xác định trên
, thỏa mãn bất
4
phương trình hàm (*) thì ta ln có f ( x )  , x  .
3
Từ (*) thay x bởi

3x
ta được:
4

3
3
9
3 f ( x )  1  3 f ( x )  f ( x ), x 

4
4
4
3
3
9
3
9
 3 f ( x )  1  3 f ( x )  f ( x )  2 3 f ( x )  f ( x ), x 
4
4
4
4
4


9
3
9
f ( x )  1  2 3 f ( x )  f ( x ), x 
4
4
4

 f ( x) 

4 8
3
9


3 f ( x )  f ( x ), x 
9 9
4
4

4
Từ đây suy ra: f ( x )  , x 
9

(1)

(2)

4
9

8
3
9
3 f ( x )  f ( x ), x 
9
4
4
8
16 64
3
9
Kết hợp (1) suy ra: f 2 ( x )  f ( x )   3 f ( x )  f ( x )  , x 
9
81 81 

4
4


Từ (2) có: f ( x )  

skkn


 f 2 ( x) 

8
16 64
3
f ( x )   .3 f ( x )  0, x 
9
81 81
4
2

4  64
3

  f ( x )    .3 f ( x ), x 
9
81
4

4
4 8

3x
Vậy f ( x )  , x  và f ( x )    3 f ( ), x  .
9 9
4
9
4
4 8
Ta xây dựng dãy số (un ) như sau: u1  ; un1    3un , n  1,2,... Khi đó
9
9 9
4
un  , n  1,2,... Ta có f ( x)  u1 (x  ). Giả sử f ( x)  un , x  , khi đó:
9
4 8
3x
4 8
f ( x)   
3f ( )   
3un  un 1 , x 
9 9
4
9 9
Theo nguyên lý quy nạp suy ra: f ( x)  un , n  1,2,..., x 

Cũng bằng nguyên lý quy nạp ta chứng minh được dãy (un ) tăng và bị chặn trên (bởi số
4
4
4
) nên có giới hạn. Suy ra lim un  . Từ đó: f ( x )  lim f ( x )  lim un  , x 
n

n
n
3
3
3
4
Vậy số k lớn nhất phải tìm là .
3
Bài tốn 2.1.1.17. [Bulgaria, 1998]. Chứng minh rằng không tồn tại hàm số
f : + ® + thỏa mãn

f 2 (x ) ³ f (x + y)éëf (x ) + yù
(1)
û
với mọi cặp số thực dương x, y .
Lời giải 1. Giả sử tồn tại hàm số f như trên. Khi đó, ta có thể viết lại giả thiết dưới dạng
f 2 (x ) + yf (x )- yf (x ) ³ f (x + y)éëf (x )+ yù
û,
hay

éf (x ) + yùéf (x )- f (x + y)ù³ yf (x ).
ë
ûë
û

Từ đây ta có

f (x )- f (x + y) ³
với mọi x , y Ỵ
ta cố định x0 Ỵ


+
+

yf (x )
f (x ) + y

. Kết quả trên chứng tỏ f là một hàm giảm thực sự trên

+

. Bây giờ,

và chọn số tự nhiên n sao cho nf (x + 1) 1 . Khi ú, ta cú


f ỗỗx +
ỗố



kử
ỗỗx + k + 1ữ
f ỗỗx + ữ
f






ỗố

ốỗ
nứ
n ứ
f ỗỗx +
ỗố


kữ



nữ

kữ
+


nữ

1 1
.
n
n = 1
³
1 1 2n
+
n n


skkn

1
f (x + 1)´
n
>
1
f (x + 1) +
n

1
n
1
n


với mọi k Ỵ

, k < n . (Chú ý rằng hàm số g (t ) =

t
với u > 0 là hàm tăng trên
t+ u

+

).
Trong bất đẳng thức trên, lần lượt cho k nhận giá trị từ 0 đến n - 1 và cộng tất cả các
bất đẳng thức lại theo vế. Khi đó, ta thu được kết quả sau


1
2

f (x )- f (x + 1) ³

(2)

với mọi x > 0 . Cố định x > 0 ,bây giờ, chọn số tự nhiên m sao cho m ³ 2 f (x ), ta có

m
+ éëf (x + m - 1)- f (m)ù
>
û 2 ³ f (x ),
Từ đó suy ra f (x + m) < 0 (mâu thuẫn). Vậy không tồn tại hàm f nào thỏa mãn (1).
f (x )- f (x + m) = éëf (x )- f (x + 1)ù
û+

Lời giải 2.
Thay y = f (x ) vào (1) ta thu được

f (x + f (x ))£

f (x )

2
với mọi x > 0 . Trong (3) ta thay x bởi x + f (x ) thì có
f (x + f (x ) + f (x + f (x )))£

(3)


f (x + f (x ))
2

£

f (x )
22

với mọi x > 0 . Mặt khác, cũng theo (3), ta có

x + f (x ) + f (x + f (x ))£ x + f (x )+

f (x )
2

.

Từ đó, sử dụng tính nghịch biến (như trong lời giải 1) của hm f , ta thu c


f ỗỗx +
ỗố
vi mi x > 0 v n ẻ

ổ 1
ỗỗ1 + +
ỗố 2
1
*
. Do 1 + +

2

ư÷ f (x )
ư

f
x
÷£
(
)
÷
ø
2n ÷
ø÷ 2 n+ 1
1
1
+ n = 2 - n < 2 và f giảm nên từ trên,
2
2

+

ta suy ra

f (x + 2. f (x ))£
với mọi x > 0 và n Ỵ

*

f (x )

2 n+ 1

. Trong bất đẳng thức trên, ta cho n ® + ¥ thì thu được

f (x + 2. f (x ))£ 0 .
Mâu thuẫn vì f ln nhận giá trị dương.

® + thỏa mãn
f ( f (x )) = 6 x - f (x )

Bài tốn 2.1.1.18. Tìm tất cả các hàm số f :

+

với mọi số thực dương x .
Lời giải. Cố định x > 0 và đặt f0 (x ) = x , fn (x ) = f ( fn- 1 (x )). Từ giả thiết, bằng
cách sử dụng quy nạp, ta chứng minh được

skkn


fn+ 2 (x ) + fn+ 1 (x )- 6 fn (x ) = 0
với mọi n Ỵ * . Đây là phương trình sai phân tuyến tính cấp hai nên bằng cách xét
phương trình đặc trưng của nó, ta tìm được cơng thức tổng qt của fn (x ) là

fn (x ) =
với mọi n Ỵ

*


2 x - f (x )
5

n

´ (- 3) +

3x + f (x )
5

´ 2n

. Do fn (x ) > 0 nên kết quả trên, ta suy ra
n

với mọi n Ỵ

ỉ ư
é2 x - f (x )ựỗỗ- 3 ữ


ỷỗố 2 ứ
ữ + 3x + f (x )> 0
*
. Nếu 2 x - f (x ) > 0 thì bằng cách xét n lẻ, n = 2k + 1 v cho

k đ + Ơ , ta có thể thấy bất đẳng thức trên sẽ khơng thể ln đúng. Cịn nếu
2 x - f (x ) < 0 thì bằng cách xét n chẵn, n = 2 k v cho k đ + Ơ , ta cng thu được
kết luận tương tự. Do vậy, ta phải có f (x ) = 2 x .
Hàm này thỏa mãn các yêu cầu của bài toán.

Nhận xét. Bài toán trên cũng có thể được giải bằng phương pháp kẹp dãy số như sau: Xét
hai dãy số (an ) và (bn ) được xác định bởi a0 = 0, b0 = 6 và

an+ 1 =

6
6
, bn+ 1 =
.
1 + bn
1 + an

Dễ thấy an , bn > 0 với mọi n ³ 1 . Ta có

an+ 2 =

6
=
1 + bn+ 1

6
6
1+
1 + an

=

6 (1 + an )
an + 7


.

Do đó

an+ 2 - 2 =

4 an - 2
an + 7

£

4
an - 2 .
7

Từ đây, bằng cách sử dụng nguyên lý ánh xạ co, ta chứng minh được
lim a2 n = lim a2 n+ 1 = 2 , hay lim an = 2 . Tương tự, ta cũng có lim bn = 2 .
Bây giờ, ta sẽ chứng minh bằng quy nạp rằng

an x < f (x ) < bn x
với mọi n Î

*

(1)

. Do f ( f (x ))> 0 nên 6 x - f (x )> 0 , từ đây ta dễ thấy khẳng định

đúng với n = 1. Giả sử khẳng định đúng với n = k , tức ta có


ak x < f (x ) < bk x
Khi đó, thay x bởi f (x ) vào bất đẳng thức trên và sử dụng giả thiết, ta thu được

ak f (x ) < f ( f (x ))< bk f (x )
Hay

ak f (x ) < 6 x - f (x ) < bk f (x ).
Từ đây, ta dễ dàng suy ra

skkn


6
6
x < f (x ) <
x = bk + 1 x .
1 + bk
1 + ak
Như vậy, khẳng định cũng đúng với n = k + 1. Bất đẳng thức (1) được chứng minh. Bây
giờ, trong (1), bằng cách cho n đ + Ơ v chỳ ý lim an = lim bn = 2 như đã chứng
minh ở trên, ta dễ dàng suy ra f (x ) = 2 x với mọi x > 0 .
ak + 1 x =

Bài toán 2.1.1.19.(Bulgaria 2006) Cho hàm số f :  0;     0;   thỏa mãn điều kiện
f  x  y   f  x  y   4 f  x  f  y  , x  y  0

a) Chứng minh rằng f  2 x   4 f  x  , x  0 ;
b) Tìm tất cả hàm số f thỏa mãn điều kiện đã cho.
Lời giải: Để chứng minh đẳng thức f  2 x   4 f  x  , x  0 ta đi theo hướng sau: Ta sẽ
tính các f  nx  , n  3 theo f  x  , f  2 x  sau đó tìm một số ngun dương n mà có hai

cách biểu diễn được theo f  x  , f  2 x  . Từ đó ta sẽ được đẳng thức liên quan giữa f  x 
và f  2 x  . Để thực hiện được điều này ta sẽ lần lượt tính f  3x  , f  4 x  , f  5 x  ,...
Đặt a  f  x  , b  f  2 x  . Khi đó kết hợp với phương trình đã cho ta có
f  2 x  x   f  2 x  x   4 f  2 x  f  x   f  3x   a  4 ab (1)



f  3x  x   f  3x  x   4 f  3x  f  x   f  4 x   b  4 a a  4 ab



(2)

Chú ý ta có thể viết 5 x  4 x  x hoặc 5 x  3x  2 x nên ta có thể tính f  5 x  theo hai
cách khác nhau
f  4 x  x   f  4 x  x   4 f  4 x  f  x   f  5 x   f  3x   4 f  4 x  f  x 





 f  5 x   a  4 ab  4 a  b  4 a a  4 ab  (3)



f  3x  2 x   f  3x  2 x   4 f  3x  f  2 x   f  5 x   f  x   4 f  3x  f  2 x 






 f  5 x   a  4 b a  4 ab (4)

Từ (3) và (4) thu được







a  4 ab  4 a  b  4 a a  4 ab   a  4 b a  4 ab









 ab  a  b  4 a a  4 ab   b a  4 ab


















 ab  a  b  4 a a  4 ab   2 ab.a  b  4 a a  4 ab   b a  4 ab













 a  b  4 a a  4 ab   2 ab.a  b  4 a a  4 ab   4b ab




2




 a   b  4 a a  4 ab   2 ab.a  b  4 a a  4 ab    16ab3




2







skkn






2



 a   b  4 a a  4 ab   2 ab.a  b  4 a a  4 ab    16b3 (5)





1
+) Nếu a  b thì
4











a   b  4 a a  4 ab   2 ab.a  b  4 a a  4 ab  













2


2





1

1  
1 1
1
1
1
1
1
 b b  4 b  b  4 b.b    2 b.b. b  b  4 b  b  4 b.b    16b 3

4  
4 4
4
4
4 
4 4
4
 
  





vơ lí với (5).
1
4

+) Nếu a  b thì


a   b  4 a a  4 ab   2 ab.a  b  4 a a  4 ab  













2

2






1

1  
1 1
1
1
1
1
1
 b b  4 b  b  4 b.b    2 b.b. b  b  4 b  b  4 b.b    16b 3

4  
4 4
4
4
4 
4 4
4
 
  




vơ lí với (5).
1
4

Do đó a  b  f  2 x   4 f  x  , x  0 .
b) Từ phương trình ban đầu ta được

f

  n  1 x   f   n  1 x   4 f  nx  f  x  , x  0, n 

, n  1 (6)

Kết hợp với f  2 x   4 f  x  , x  0 và sử dụng phương pháp quy nạp ta được
f  nx   n 2 f  x  , n  1, x  0 (7).
Từ (7) ta xác định được f  x  với x 
được
 1
f 1  f  n.   n 2 f
 n

Với m, n 

*

 0;   . Thật vậy, trong (7) thay

x

1
ta
n

1
 1  f 1
  f   2
n

n
n

ta có

f 1  m 
m
 1
1
m m
f    f  m.   m 2 f    m 2 . 2    f 1  f      f 1 (7)
n
n
 n
n
n
n n
2
Từ (7) suy ra f  r   r f 1 , r 
 0;   (8).
2

2

Theo kết quả cơ bản đã biết nếu có đẳng thức (8) và hàm số f đơn điệu thì ta
được f  x   x 2 f 1 , x   0;   . Thật vậy, đầu tiên ta chỉ ra hàm số f đơn điệu
tăng. Từ giả thiết ta có
f  x  y   f  x  y   4 f  x  f  y   0, x  y  0
 f  x  y   f  x  y  , x, y   0;   , x  y


u  x  y
v  x  y

Với u  v  0 , tồn tại các số dương x  y  0 sao cho 

skkn


Suy ra f  x  y   f  x  y  , x, y   0;  
 f  u   f  v  , u, v   0;   , u  v hay hàm số f đồng biến.
Dựa theo tính trù mật của tập
trong
ta có, với mỗi số x  0 , tồn tại hai dãy
 un  ,  vn  hữu tỉ sao cho  un  là dãy số tăng có giới hạn là x và  vn  là dãy số
giảm có giới hạn là x .
Ta có bất đẳng thức sau
un  x  vn , n 

*

 f  un   f  x   f  vn  , n 

*

 f 1 un2  f  x   f 1 vn2 , n 

*

(9)
Từ (9) chuyển qua giới hạn ta được


 f 1 x 2  f  x   f 1 x 2 , x   0;  
 f  x   f 1 x 2 , x   0;   . Thử lại ta thấy hàm f  x   ax 2 , x   0;   , trong

đó a là hằng số dương, thỏa mãn điều kiện.
Bài tốn 2.1.1.20. Tìm tất cả các hàm số f : R   R  thỏa mãn

f  f  x   y   f  x  y   2x  2 f  y 

với mọi x, y  0.
Lời giải. Đầu tiên, đổi chỗ x, y , ta có f  f  y   x   f  x  y   2 y  2 f  x  . Suy ra
f  f  x   y   f  f  y   x   2  x  y   2  f  y   f  x   . 1

Trong giả thiết, thay y  f  y  , ta được
f  f  x   f  y   f  x  f  y   2x  2 f  f  y .

Đổi chỗ x, y, ta có
Suy ra

f  f  x   f  y   f  y  f  x   2 y  2 f  f  x .





f  y  f  x   f  x  f  y   2  y  x   2 f  f  x   f  f  y  . 2

Kết hợp (1) và (2), ta được

f  f  x   f  f  y   f  y   f  x   2  x  y 


hay f  f  x    f  x   2 x  f  f  y    f  y   2 y với mọi x, y  0. Do đó, tồn tại hằng số c
sao cho f  f  x    f  x   2 x  c với mọi x. Đến đây, bằng quy nạp, ta suy ra
1
c  (2) n 
c
f  n   x    f  x   2 x  nc   
  f  x  x  
3
3
3 
3
với n  2, trong đó ký hiệu f n  x   f f  x  .





n lan ham f

Với x cố định, cho n   thì lim
f  n   x   0 nên f  x   x 

f  x   x, x  0.

(2) n
  với n lẻ và ta vẫn phải có
3

c

với mọi x  0. Thay vào đề bài, ta được c  0 hay
3

skkn


×